i need y’all help asap

I Need Yall Help Asap

Answers

Answer 1

Answer:

2

Step-by-step explanation:

answer is in photo above

I Need Yall Help Asap

Related Questions

help please? <3 asap​

Answers

Answer:b

Step-by-step explanation:

Answer:

It is b because it just makes the most scence

Write an augmented matrix of each system. Then, solve each system using matrix notation. Describe the solution set in vector notation.

Answers

Answer:

The answer is below

Step-by-step explanation:

Given the system of equations:

x + y + 2z = 9

2x + 4y - 3z = 1

3x + 6y - 5z = 0

This system of equation can be solved using matrix. This done by first representing the equations as matrix and then solving:

The matrix form is:

[tex]\left[\begin{array}{ccc}1&1&2\\2&4&-3\\3&6&-5\end{array}\right] \left[\begin{array}{c}x\\y\\z\end{array}\right] = \left[\begin{array}{c}9\\1\\0\end{array}\right] \\\\\\ \left[\begin{array}{c}x\\y\\z\end{array}\right] =\left[\begin{array}{ccc}1&1&2\\2&4&-3\\3&6&-5\end{array}\right] ^{-1} \left[\begin{array}{c}9\\1\\0\end{array}\right] \\\\\\[/tex]

[tex]\left[\begin{array}{c}x\\y\\z\end{array}\right] =\left[\begin{array}{ccc}2&-17&11\\-1&11&-7\\0&3&-2\end{array}\right] \left[\begin{array}{c}9\\1\\0\end{array}\right] \\\\\\ \left[\begin{array}{c}x\\y\\z\end{array}\right] =\left[\begin{array}{c}1\\2\\3\end{array}\right][/tex]

A certain group of test subjects had pulse rates with a mean of 75.2 beats per minute and a standard deviation of 11.2 beats per minute. Use the range rule of thumb to identify the limits separating values that are significantly low or significantly high. Is a pulse rate of 147.6 beats per minute significantly low or significantly​ high?

Answers

Answer:

The z-score for a pulse rate of 147.6 beats per minute is 6.46 > 2, which means that this pulse rate is significantly high.

Step-by-step explanation:

Normal Probability Distribution:

Problems of normal distributions can be solved using the z-score formula.

In a set with mean [tex]\mu[/tex] and standard deviation [tex]\sigma[/tex], the z-score of a measure X is given by:

[tex]Z = \frac{X - \mu}{\sigma}[/tex]

The Z-score measures how many standard deviations the measure is from the mean. After finding the Z-score, we look at the z-score table and find the p-value associated with this z-score. This p-value is the probability that the value of the measure is smaller than X, that is, the percentile of X. Subtracting 1 by the p-value, we get the probability that the value of the measure is greater than X.

By the Range Rule of Thumb, if Z < -2, the measure X is significantly low, and if Z > 2, the measure X is significantly high.

Mean of 75.2 beats per minute and a standard deviation of 11.2 beats per minute.

This means that [tex]\mu = 75.2, \sigma = 11.2[/tex]

Is a pulse rate of 147.6 beats per minute significantly low or significantly​ high?

We have to find Z when X = 147.6. So

[tex]Z = \frac{X - \mu}{\sigma}[/tex]

[tex]Z = \frac{147.6 - 75.2}{11.2}[/tex]

[tex]Z = 6.46[/tex]

The z-score for a pulse rate of 147.6 beats per minute is 6.46 > 2, which means that this pulse rate is significantly high.

)Which statement best describes the area of the triangle shown below?

A coordinate grid is shown with a triangle.. The base is 4 units, and the height is 4 units.

It is one-half the area of a rectangle of length 4 units and width 2 units.
It is one-half the area of a square of side length 4 units.
It is twice the area of a rectangle of length 4 units and width 2 units.
It is twice the area of a square of side length 4 units.

Answers

9514 1404 393

Answer:

  (b)  It is one-half the area of a square of side length 4 units.

Step-by-step explanation:

The triangle described can be formed by drawing the diagonal through a square 4 units on a side. Such a diagonal cuts the square cleanly in half, so the area of the triangle is half the area of the square.

Answer:

B

Step-by-step explanation:

Hope you have a nice day! :o)

please helppppp
i am v stuck

Answers

Answer:

Below

Step-by-step explanation:

First find the area of the triangle

A = bh/2

   = (5)(10)/2

   = 50/2

   = 25 in^2

*the height of the triangle is 10 because we are disregarding the 3 inches as it is not a part of the triangle

You can now find the volume of the triangular part

V = area x height

   = (25)(10)

   = 250 in^3

Now find the volume of the rectangular part

V = LWH

   = (10)(5)(3)

   = 150 in^3

Add these two up to get the total volume

150+250 =400 in^3

I hope that is correct.... good luck!

Determine if the specified linear transformation is ​(a​) ​one-to-one and ​(b​) onto. Justify your answer. ​T: ​, ​T(​)​(​,​), ​T(​)​(​,​), and ​T(​)​(​,​), where ​, ​, are the columns of the 33 identity matrix. a. Is the linear transformation​ one-to-one? A. T is​ one-to-one because ​T(x​)0 has only the trivial solution. B. T is not​ one-to-one because the columns of the standard matrix A are linearly independent. C. T is not​ one-to-one because the standard matrix A has a free variable. D. T is​ one-to-one because the column vectors are not scalar multiples of each other. b. Is the linear transformation​ onto? A. T is onto because the columns of the standard matrix A span . B. T is not onto because the columns of the standard matrix A span . C. T is onto because the standard matrix A does not have a pivot position for every row. D. T is not onto because the standard matrix A contains a row of zeros.

Answers

Answer:

C. T is not​ one-to-one because the standard matrix A has a free variable.

Step-by-step explanation:

Given

[tex]T(x_1,x_2,x_3) = (x_1-5x_2+4x_3,x_2 - 6x_3)[/tex]

Required

Determine if it is linear or onto

Represent the above as a matrix.

[tex]T(x_1,x_2,x_3) = \left[\begin{array}{ccc}1&-5&4\\0&1&-6\\0&0&0\end{array}\right] \left[\begin{array}{c}x_1&x_2&x_3\end{array}\right][/tex]

From the above matrix, we observe that the matrix does not have a pivot in every column.

This means that the column are not linearly independent, & it has a free variable and as such T is not one-on-one

ANSWER ASAP!! Use the substitution method to solve the system of equations. 3x+7y=1 and y=x-7

Answers

Answer:

x=5

y = -2

(5,-2)

Step-by-step explanation:

3x+7y=1 and y=x-7

Substitute the second equation in for y in the first equation

3x+7( x-7)=1

Distribute

3x+ 7x - 49 = 1

Combine like terms

10x - 49 =1

Add 49 to each side

10x-49+49 = 1+49

10x = 50

Divide by 10

10x/10 = 50/10

x = 5

Now find y

y = x-7

y = 5-7

y = -2

What is the slope of a line perpendicular to the line whose equation is x + 2y = 2. Fully simplify your answer. ​

Answers

9514 1404 393

Answer:

  2

Step-by-step explanation:

We can solve for y to find the slope of the given line.

  x +2y = 2

  2y = -x +2 . . . . . subtract x

  y = -1/2x +1 . . . . divide by the coefficient of y

The slope is the x-coefficient, -1/2.

Any perpendicular line will have a slope that is the opposite reciprocal of this:

  -1/(-1/2) = 2

A perpendicular line will have a slope of 2.

Part
The number 3.14 is a rounded value of the mathematical quantity . If you replace 3.14 with 1, what is the exact formula for the circumference in
terms of the diameter, d, and in terms of the radius, r?
B IV x x
Font Sizes
А
E1 E2
Characters used: 0/15000
Submit
I need help

Answers

Answer:

C = 2(3.14)r

C = (3.14)d

Step-by-step explanation:

Circumference in terms of radius:

[tex]c \: = 2\pi \: r[/tex]

So replacing pi with 3.14 you would get 2(3.14)r

Circumference in terms of diameter:

[tex]c = d\pi[/tex]

So replacing pi with 3.14 you would get d(3.14)

2x^2 + 3x - 12 when x = 5 help pls

Answers

2x^2 + 3(5)-12
2x^2+ 15+-12

combine like terms

2x^2+15+-12
(2x^2)+(15+-12)

Answer:
2x^2 + 3

[tex]\huge\textsf{Hey there!}[/tex]

[tex]\mathsf{2x^2 + 3x - 12}[/tex]

[tex]\mathsf{= 2(5)^2 + 3(5) - 12}[/tex]

[tex]\mathsf{5^2}[/tex]

[tex]\mathsf{= 5 \times 5}[/tex]

[tex]\mathsf{\bf = 25}[/tex]

[tex]\mathsf{2(25) + 3(5) - 12}[/tex]

[tex]\mathsf{2(25)}[/tex]

[tex]\mathsf{= \bf 50}[/tex]

[tex]\mathsf{3(5)}[/tex]

[tex]\mathsf{= \bf 15}[/tex]

[tex]\mathsf{= 50 + 15 - 12}[/tex]

[tex]\mathsf{50 + 15}[/tex]

[tex]\mathsf{= \bf 65}[/tex]

65 - 12

[tex]\mathsf{= \bf 53}[/tex]

[tex]\boxed{\boxed{\huge\textsf{Answer: \bf 53}}}\huge\checkmark[/tex]

[tex]\large\textsf{Good luck on your assignment and enjoy your day!}[/tex]

~[tex]\frak{Amphitrite1040:)}[/tex]

Helppppppppp

Which choice is equivalent to the product below for acceptable values of X?

Answers

Answer:

C is the answer

to solve this, you just have to multiply the values inside the square roo, and the 6x will be distributed to each values inside the other radical.

Note: You can multiply the radicals because they both have a square root.

The value of the equivalent equation is A = √ ( 6x² + 18x )

What is an Equation?

Equations are mathematical statements with two algebraic expressions flanking the equals (=) sign on either side.

It demonstrates the equality of the relationship between the expressions printed on the left and right sides.

Coefficients, variables, operators, constants, terms, expressions, and the equal to sign are some of the components of an equation. The "=" sign and terms on both sides must always be present when writing an equation.

Given data ,

Let the equation be represented as A

Now , the value of A is

Let the first variable be p = √6x

Let the second variable be q = √ ( x + 3 )

Now , the value of A = pq

Substituting the values in the equation , we get

A = √6x [ √ ( x + 3 ) ]

On simplifying the equation , we get

A = √6x ( x + 3 )

A = √ ( 6x² + 18x )

Hence , the equation is A = √ ( 6x² + 18x )

To learn more about equation click :

https://brainly.com/question/19297665

#SPJ7

Please explain or answer this! Answer getting brainliest!

Answers

The correct answer is c
Yes I agree the correct answer is c

GCF of 9x^2y^2 and 5x^2y^3

Answers

Answer:

x^2y^2.

Step-by-step explanation:

1/2n+3=6 what is the solution to this equation

Answers

Answer:

n=6

Step-by-step explanation:

1/2 n + 3 = 6

Subtract 3 from each side

1/2 n +3-3=6-3

1/2n = 3

Multiply each side by 2

1/2 n * 2 = 3*2

n = 6

Given that,

1/2n + 3 = 6

Now we can subtract 3 from each side,

→ 1/2 n +3-3 = 6-3

→ 1/2n = 3

Now we can multiply each side by 2,

→ 1/2 n × 2 = 3 × 2

→ n = 6

Hence, 6 is the correct answer.

Each side a square calendar is 7 inches long. what is the calendar's area

Answers

Answer:

49 square inches

Step-by-step explanation:

Length x width =area

since it is a square, all sides have a length of 7 inches

7x7=49 inches ^2

Telephone poles are stored in a pile with 30 poles in the first layer, 29 in the second, and so on. If there are 12 layers, how many telephone poles does the pile contain?

Answers

Answer: 294

Step-by-step explanation:

Given

First layer has 30 Poles

Second layer has 29 Poles

There are twelve layers

It follows an A.P. with first term [tex]a_1=30[/tex] and common difference [tex]d=-1[/tex]

Sum of n terms of an A.P. is

[tex]\Rightarrow S_n=\dfrac{n}{2}[2a+(n-1)d][/tex]

Insert the values

[tex]\Rightarrow S_n=\dfrac{12}{2}[2\times 30+(12-1)(-1)]\\\\\Rightarrow S_n=6[60-11]\\\Rightarrow S_n=294[/tex]

So, there are 294 Poles in 12 layers

What is the MEDIAN or Q2 of the data set: 24, 25, 29, 30, 31, 31, 32, 34, 34? ​

Answers

Answer:

31

Step-by-step explanation:

Median = middle number in terms of value

Usually the first step is to list the numbers in order from least to greatest however the numbers are already listed in order from least to greatest

24, 25, 29, 30, 31, 31, 32, 34, 34

We the just go to the middle number

The middle number would be 31

So we can conclude that the median is 31

f(x)=2x^2-x-6 g(x)=4-x (f+g)(x)

Answers

[tex] \large \boxed{(f + g)(x) = f(x) + g(x)}[/tex]

The property above is distribution property where we distribute x-term in the function.

Substitute both f(x) and g(x) in.

[tex] \large{ \begin{cases} f(x) = 2 {x}^{2} - x - 6 \\ g(x) = 4 - x \end{cases}} \\ \large{f(x) + g(x) = (2 {x}^{2} - x - 6) + (4 - x)} \\ \large{f(x) + g(x) = 2 {x}^{2} - x - 6 + 4 - x}[/tex]

Évaluate/Combine like terms.

[tex] \large{f(x) + g(x) = 2 {x}^{2} - 2x - 2} [/tex]

The function can be factored so there are two answers. (Both of them work as one of them is factored form while the other one is not.)

[tex] \large{(f + g)(x) = 2 {x}^{2} -2x -2}[/tex]

Alternative

[tex] \large{(f + g)(x) = 2({x}^{2} -x - 1)}[/tex]

Answer

(f+g)(x) = 2x²-2x-2(f+g)(x) = 2(x²-x-1)

Both answers work. The second answer is in factored form.

Let me know if you have any doubts!

The base of a triangle is 9cm correct to the nearest cm.
The area of this triangle is 40 cm2 correct to the nearest 5 cm?.
Calculate the upper bound for the perpendicular height of this triangle.

Answers

Answer:

can you show the picture plz

PLEASE HELP ME I WILL MARK TOU JT YOU HELP ME!!!

Answers

Given:

The figure of a construction.

To find:

The correct option that represents the given construction.

Solution:

In the given figure, the given angle is angle CAB.

The steps for the given figure are:

1. Draw a ray PQ.

2. Mark an arc BC on the given angle and mark the same arc on the line and the intersection of line and arc is point Q.

3. Set the compass of the length BA.

4. Put the compass on the point Q mark arc that intersect the first arc. The intersection of arcs is R.

5. Draw a ray PR.

These are the steps of construction to copy an angle.

Therefore, the correct option is A.

The first side of a triangle measures 4 in. less than the second side, the third side is 3 in more than the first side, and the perimeter is 15 in. How long is the third side?


If s represents the length of the second side, which of the following represents the length of the third side?
O S-4
OS-1
OS+3

Answers

Answer:

s-1

Step-by-step explanation:

1st side:  s-4

2nd side: s-4+3, which is 's-1'

Elena receives ​$131 per year in simple interest from three investments totaling ​$3000. Part is invested at​ 3%, part at​ 4% and part at​ 5%. There is ​$1000 more invested at​ 5% than at​ 4%. Find the amount invested at each rate.
The amount invested at​ 3% is ​$___ the amount invested at​ 4% is ​$___ , and the amount invested at​ 5% is ​$___

Answers

Answer:

Elena invested $ 1,700 at 5%, $ 700 at 4%, and $ 600 at 3%.

Step-by-step explanation:

Given that Elena receives $ 131 per year in simple interest from three investments totaling $ 3000, and part is invested at 3%, part at 4% and part at 5%, and there is $ 1000 more invested at 5% than at 4%, to find the amount invested at each rate, the following calculations must be performed:

1500 x 0.05 + 500 x 0.04 + 1000 x 0.03 = 75 + 20 + 30 = 125

1600 x 0.05 + 600 x 0.04 + 800 x 0.03 = 80 + 24 + 24 = 128

1700 x 0.05 + 700 x 0.04 + 600 x 0.03 = 85 + 28 + 18 = 131

Therefore, Elena invested $ 1,700 at 5%, $ 700 at 4%, and $ 600 at 3%.

In a study of reaction times to a specific stimulus, a psychologist recorded these data (in seconds). Find the variance and standard deviation. Use a TI-83 Plus/TI-84 Plus calculator and round the answers to at least two decimal places.

Answers

Answer:

s² = 0.7965

s = 0.8925

Step-by-step explanation:

Given the data :

Class limit __ midpoint(x) ___ Frequency

2.1 - 2.7 ______2.4 ________13

2.8 - 3.4 ______3.2 _______ 14

3.5 - 4.1 ______ 3.8 _______ 10

4.2 - 4.8 ______4.5 _______ 6

4.9 - 5.5______ 5.2 ________2

5.6 - 6.2______ 5.9 ________ 1

The midpoint was calculated thus :

(Upper limit + lower limit) / 2

Using a graphing calculator :

The sample variance, s² = 0.7965

The sample standard deviation, s = 0.8925

A rectangular room has a perimeter of 70m what could be the length of the longest side of the room​

Answers

Answer:

34

Step-by-step explanation:

70-2=68

side 1 & 3=1m each

68 divided by 2 = 34

side 2 & 4 = 34m each

Choose all the points that would be acceptable points to choose to test for shading for −3(m+7)≥−24

Answers

Answer:

m≤1

Step-by-step explanation:

The given inequality is :

−3(m+7)≥−24

Dividing both sides by -3.

(m+7) ≤ 8

Subtract 7 from both sides,

(m+7)-7 ≤ 8-7

m≤1

Hence, this is the required solution.

A fire ranger stands atop an observation tower 70 feet above the ground and sees a fire in the distance. She measures the angle of depression from where she is to the fire and finds it to be 34° How far away from the base of the tower is the fire?​

Answers

Answer:

[tex]103.78\:\mathrm{ft}[/tex]

Step-by-step explanation:

We can form a right triangle where the distance between the ranger's current position and fire is the hypotenuse of the triangle. In a right triangle, the tangent of an angle is equal to its opposite side divided by the hypotenuse.

Therefore, we have:

[tex]\tan 34^{\circ}=\frac{70}{x}[/tex], where [tex]x[/tex] is the distance between the base of the tower and the fire.

Solving, we get:

[tex]x=\frac{70}{\tan 34^{\circ}}=103.779267796\approx \boxed{103.78\:\mathrm{ft}}[/tex]

can someone help me solve this pls ? solve |x| + 7 < 4

Answers

Answer:

no solution

Step-by-step explanation:

|x| + 7 < 4

Subtract 7 from each side

|x| + 7-7 < 4-7

|x| < -3

But an absolute value must be greater than or equal to zero

There is no solution

Answer: No solutions

Let's solve the inequality step-by-step

[tex]|x|+7<4[/tex]

First subtract 7 from both sides

[tex]|x|+7-7<4-7\\|x|<-3[/tex]

Now usually you'd solve absolute value, but absolute value cannot be less than 0 so there can't be a solution.

3.
June has 248 oranges. She sells of these oranges. How many oranges did June sell?
Answer.​

Answers

Answer:

93. Divide 248 by 8. One eighth of 248 = 31.

June sold 3/8 of the 248 oranges.

If 1/8 = 31 and June sold 3/8 of the oranges.

Multiply 3 × 31. Therefore June sold 93 oranges.

Step-by-step explanation:

Answer:

93 oranges

Step-by-step explanation:

3/8 of 248

[tex]\frac{y}{248} :\frac{\frac{3}{8} }{1}[/tex]

y × 1 = 248 × 3/8

y = 93

which exponential equation is equivalent to the logarithmic equation below? log478=a
A. 478^10 = a
B. a^10 = 478
C. 478^a = 10
D. 10^a = 478

Answers

I think the answer is D

Can somone help me solve this

Answers

Answer:

solid shade above

Step-by-step explanation:

graph line same way you would if it was an equal sign

all the y's that are greater are above the line

its solid because is greater than OR equal to. Equal to includes the line itself as a set of solutions

Other Questions
Answers include A.5/13B -5/13C 5D -5 To work with electrical wires, extreme caution should be used.Which revision most effectively corrects the dangling modifier in the sentence? wriite an article for yourschool magazine 1 paragrapgh.analyzing the advantage and disadvantage of waering school uniform. I really need help or Im gonna fail the semester Joe has 1/2 of a bag of peanuts. He wants to share them equally among himself and 3 friends. How much of the bag of peanuts would each person get? A. 6B. 1/6C. 3/2D. 1 1/2Thank you!!!!! Mrs. Hopewell, who had divorced her husband long ago, needed someone to walk over the fields with her; and when Joy had to be impressed for these services, her remarks were usually so ugly and her face so glum that Mrs. Hopewell would say, "If you can't come pleasantly, I don't want you at all," to which the girl, standing square and rigid-shouldered with her neck thrust slightly forward, would reply, "If you want me, here I amLIKE I AM." Based on Mrs. Hopewell's attitude toward the fields, it is reasonable to infer that the story takes place on a farm with many field hands and employees. on a wide, isolated expanse of farmland. in a small town where everyone is friendly toward one another. in a deserted town in which mysterious events occur. A state of economic scarcity exists when consumers:A. borrow too much money to buy unnecessary products.OB. have their economic freedoms restricted by the government.O C. do not have enough resources to satisfy all of their wants.O D. save their money in banks instead of spending it.SUBS Match each example to the correct type of conflict. PLEASE HELP ME I HAVE TO GET THIS DONE BEFORE IN 10 MINUTES, ILL GIVE BRAINLIEST Determine whether the triangles are similar. If they are, find the scale factor of A ABC to A DEF. B E 12 9 64 D F 12 64 16 I need the answer fast its due in 5 minutes! What should you do if you find so much information you can't get it all into your paper's length requirements?Narrow your focus to a more specific part of your topic.Change your subject and start overAsk your teacher if you can write a longer paper.Include all your topic's points but write less about each. Akbar most helped non-Muslims byending taxes they had to pay.creating a code of law.creating his own religion.paying fair wages for work. Determine the general solution of 3sin x=2tan x The legislature refused to approve funds for the military, prompting Bismarck to approach the Catholic Church for assistance. B. The legislature funded an enormous military buildup, prompting Bismarck to abandon his Realpolitik approach. C. The legislature funded an enormous military buildup, prompting Bismarck to embrace socialism as a political strategy. D. The legislature refused to approve funds for the military, prompting Bismarck to divert money approved for other purposes. Convert each of the following masses to the units requestedA. 500 liter to deciliters B. 4200 km to hm This is a picture taken during WWII. (image) What form of genocide, as defined by the UN, does this picture most likely represent?-people living in refugee camps-the denial of food and medicine-concentration camp imprisonment -removing children to be raised by others I HAVE ONE WEEK OF SCHOOL CAN SOMEONE HELP?! Larry stylinson pictures for LarriesIm sorry if you understood the photos free P. O I N T S Can you Please Answer, This question Min and Evan played a video game. Min scored 6,978 points. This is 3 times the number of points that Evan scored.How many more points did Min score than Evan? What is special about a brainliest?